Linear Combinations in 2-space

Click For Summary
The discussion revolves around understanding the linear combinations of the vectors [2, 1] and [0, 1] in the xy-plane, specifically for values of α and β ranging from 0 to 2. Participants clarify that there are nine combinations, derived from the three possible values for α and β, which can be visualized as points on a lattice. One contributor expresses confusion about the problem's instructions and the calculation of combinations, prompting further exploration of permutations. The importance of correctly interpreting the variables and their combinations is emphasized, leading to a better understanding of the task. Ultimately, the discussion highlights the connection between the values of α and β and their resulting linear combinations.
Destroxia
Messages
204
Reaction score
7

Homework Statement



In the xy-plane mark all nine of these linear combinations:

## α \lbrack 2, 1 \rbrack + β \lbrack 0, 1 \rbrack## with c = 0, 1, 2 and d = 0, 1, 2

Homework Equations



ANSWER:

The nine combinations will lie on a lattice. If we took all whole numbers c and d, the lattice would
lie over the whole plane.

The Attempt at a Solution



I think my biggest problem in this, is not actually knowing what the question is asking.

I've tried plotting the linear combinations of me filling in α and β with the numbers listed for c & d.

1. ## 0 \lbrack 2, 1 \rbrack + 0\lbrack 0, 1 \rbrack ## yields, ## \lbrack 0, 0 \rbrack ##

2. ## 1 \lbrack 2, 1 \rbrack + 1\lbrack 0, 1 \rbrack ## yields, ## \lbrack 2, 2 \rbrack ##

3. ## 2 \lbrack 2, 1 \rbrack + 2\lbrack 0, 1 \rbrack ## yields, ## \lbrack 4, 4 \rbrack ##

I'm not sure where they are getting 9 linear combinations, but I'm pretty sure I'm misunderstanding the instructions of the problem.
 
Physics news on Phys.org
Hint: ##3 \times 3 = 9##
 
vela said:
Hint: ##3 \times 3 = 9##

Okay, so the only thing I could think of using that hint would be that I can do other permutations of the c = 0, 1, 2 and d = 0, 1, 2?

So maybe do 1 and 0, or 2 and 1, etc?
 
RyanTAsher said:

Homework Statement



In the xy-plane mark all nine of these linear combinations:

## α \lbrack 2, 1 \rbrack + β \lbrack 0, 1 \rbrack## with c = 0, 1, 2 and d = 0, 1, 2
Seems like it shoud be "with α = 0, 1, and 2, and β = 0, 1, and 2"

BTW, it's much simpler to just use the [ and ] characters than typing lbrack and rbrack in LaTeX.
RyanTAsher said:

Homework Equations



ANSWER:

The nine combinations will lie on a lattice. If we took all whole numbers c and d, the lattice would
lie over the whole plane.

The Attempt at a Solution



I think my biggest problem in this, is not actually knowing what the question is asking.

I've tried plotting the linear combinations of me filling in α and β with the numbers listed for c & d.

1. ## 0 \lbrack 2, 1 \rbrack + 0\lbrack 0, 1 \rbrack ## yields, ## \lbrack 0, 0 \rbrack ##

2. ## 1 \lbrack 2, 1 \rbrack + 1\lbrack 0, 1 \rbrack ## yields, ## \lbrack 2, 2 \rbrack ##

3. ## 2 \lbrack 2, 1 \rbrack + 2\lbrack 0, 1 \rbrack ## yields, ## \lbrack 4, 4 \rbrack ##

I'm not sure where they are getting 9 linear combinations, but I'm pretty sure I'm misunderstanding the instructions of the problem.
 
RyanTAsher said:
Okay, so the only thing I could think of using that hint would be that I can do other permutations of the c = 0, 1, 2 and d = 0, 1, 2?

So maybe do 1 and 0, or 2 and 1, etc?
Each choice of c (really ##\alpha##) can be paired with one of three possible d (really ##\beta##) values. How many combinations of the two does that make?
 
Mark44 said:
Each choice of c (really ##\alpha##) can be paired with one of three possible d (really ##\beta##) values. How many combinations of the two does that make?

Yeah, that's what I was getting at. I didn't know if the points could be interchanged, but I guess there isn't really any reason why they couldn't be.
 
Question: A clock's minute hand has length 4 and its hour hand has length 3. What is the distance between the tips at the moment when it is increasing most rapidly?(Putnam Exam Question) Answer: Making assumption that both the hands moves at constant angular velocities, the answer is ## \sqrt{7} .## But don't you think this assumption is somewhat doubtful and wrong?

Similar threads

  • · Replies 2 ·
Replies
2
Views
2K
  • · Replies 5 ·
Replies
5
Views
2K
  • · Replies 2 ·
Replies
2
Views
2K
  • · Replies 13 ·
Replies
13
Views
2K
  • · Replies 2 ·
Replies
2
Views
2K
  • · Replies 8 ·
Replies
8
Views
1K
  • · Replies 5 ·
Replies
5
Views
3K
Replies
4
Views
3K
  • · Replies 21 ·
Replies
21
Views
3K
  • · Replies 3 ·
Replies
3
Views
2K